1
$\begingroup$

Suppose that $X_k(1 \leqslant k \leqslant n )$ are positive i.i.d random variable with finite mean $\mu$ and variance $\sigma^2$. Define $P_n= (\prod \limits _{k=1}^n X_k)^{\frac{1}{n}}$ $(n\geqslant1)$and $\Pi_n=\frac{P_n-\mu}{\sigma / {\sqrt{n}}}$ $(n\geqslant1)$ Find the limit of $\Pi_n$ in distribution(i.e.,$\Pi_n \rightarrow_d ?$).

There is a hint for the question: Use the Delta Method.

First I take log on both side for $P_n= (\prod \limits _{k=1}^n X_k)^{\frac{1}{n}}$ and get: $$\log(P_n)=\frac{1}{n}\sum \limits_{k=1}^n \log(X_k)$$

I honestly don't have the slightest inkling on what to do for the next steps.

$\endgroup$
3
  • $\begingroup$ Are you sure it's not the geometric mean instead of $\mu$ in $\Pi_n$ expression? $\endgroup$ Commented Nov 12, 2021 at 14:04
  • $\begingroup$ That's the reason why I feel a little bit awkward @Harnak $\endgroup$ Commented Nov 12, 2021 at 14:38
  • $\begingroup$ well, yeah, it's pretty weird. I'm sure there's an error because with geometric mean it converges, but with mean it doesn't $\endgroup$ Commented Nov 13, 2021 at 9:54

0

You must log in to answer this question.

Start asking to get answers

Find the answer to your question by asking.

Ask question

Explore related questions

See similar questions with these tags.